¿Cuál es el porcentaje de primos en $\mathbb{N}$ ?
( $\mathbb{N} := \lbrace { 1, 2, 3, \ldots \rbrace }$ un primo sólo es divisible por sí mismo y por 1 en $\mathbb{N}$ )
El porcentaje tiene que ser inferior al 50%, ya que todos los números pares (excepto el 2) no son primos. Por lo tanto, el porcentaje tiene que ser inferior a $1 - (\frac{1}{2} + (\frac{1}{3} - \frac{1}{6})) = 1 - (\frac{1}{2} + \frac{1}{6}) = 1 - (\frac{1}{2} + \frac{1}{2 \cdot 3}) = \frac{1}{3}$
Supongo que será algo así:
$$\frac{\text{primes in } \mathbb{N}}{\text{numbers in } \mathbb{N}} = 1 - \sum_{i=\text{first Prime}}^\text{primes} \frac{1}{\prod_{j=\text{first prime}}^{i\text{-th prime}} j}$$
Pero calcular este valor (exacto) va definitivamente más allá de mis habilidades matemáticas. ¿Puede alguien ayudarme?
4 votos
Google "teorema de los números primos". Y, efectivamente, es profundo.
0 votos
teorema del número primo
4 votos
@David De hecho, el resultado más débil de Chebyshev que $c \frac{x}{\log x} \leqslant \pi(x) \leqslant C \frac{x}{\log x}$ (para las constantes $0 < c < C < \infty$ ) ya dice que los primos tienen una densidad natural nula entre $\mathbb N$ . Este resultado es, por supuesto, muy poco trivial, pero no demasiado profundo; creo que los métodos de criba son lo suficientemente buenos para obtenerlo. No creo que el PO esté preguntando por la exactitud de los resultados. asintótica de la densidad...
16 votos
El hecho de que $\lim_{n \to \infty} \dfrac{\text{primes } \leq n}{n} = 0$ es mucho más fácil de derivar que el teorema de los números primos, o incluso las estimaciones de Chebyshev. El PO está en el camino correcto con sus cálculos para tamizar $2$ y $3$ .
1 votos
@moose: Tu estimación, que yo interpreto como $1-\left(\frac{1}{2}+\frac{1}{6}+\frac{1}{30}+\frac{1}{210}+\cdots\right)$ da una densidad positiva, lo que no es correcto. Tal vez se observe que la fracción atendida por $2$ , $3$ y $5$ es en realidad $(1-1/2)(1-1/3)(1-1/5)$ que es $11/15$ , mientras que su $1/2+1/6+1/30$ es $21/30$ .
2 votos
@moose: No estoy seguro de que haya cierto consenso sobre el tiempo que hay que esperar antes de aceptar una respuesta (ver meta.math.stackexchange.com/questions/2553/ ), pero creo que has aceptado demasiado pronto. La respuesta de Pete Clark es hermosa y la prueba que sugiere es realmente elemental (al menos en comparación con las pruebas sugeridas en otras respuestas hasta ahora).
1 votos
@Martin: Estimado Moose, la respuesta de Pete Clark es exactamente lo que tenía en mente con mi comentario anterior. Saludos,
0 votos
@MartinSleziak Moose puede desaceptar una respuesta y aceptar otra diferente en cualquier momento, si así lo desea.
0 votos
Véase también: Demostrar que $\lim_{x\to\infty} \pi(x)/x=0$
0 votos
Una pregunta reciente sobre MathOverflow : ¿Cuál es la prueba más sencilla de que la densidad de los primos es cero?